A man saves Rs.7,500 in first year. In each year after the first, he saves Rs. 2,000 more than he does in the preceding year. When will he be saved the total amount Rs.1,65,000? Find it.​

Answers

Answer 1

Answer:

aeqtq34t

Step-by-step explanation:

q34t35t134


Related Questions

17- 20: In an August 2012 Gallup survey of 1,012 randomly selected U.S. adults (age 18 and over), 53% said that they were dissatisfied with the quality of education students receive in kindergarten through grade 12. They also report that the "margin of sampling error is plus or minus 4%."17. What is the population of interest?18. What is the sample being used?19. What is the population parameter of interest, and what is the correct notation for this parameter? What is the relevant statistic?20. Find an interval estimate for the parameter of interest. Interpret it in terms of dissatisfaction in the quality of education students receive. Use two decimal places in your answer.

Answers

17. The population of interest is U.S. adults (age 18 and over).
18. The sample being used is 1,012 randomly selected U.S. adults (age 18 and over).
19. The population parameter of interest is the percentage of U.S. adults (age 18 and over) who are dissatisfied with the quality of education students receive in kindergarten through grade 12. The correct notation for this parameter is p and the relevant statistic is 53%.
20. The interval estimate for the parameter of interest is 49% - 57%, which indicates that between 49% and 57% of U.S. adults (age 18 and over) are dissatisfied with the quality of education students receive in kindergarten through grade 12.

How could 1/5 + -4/5 be modeled on a number line

Answers

The expression 1/5 + -4/5 has plotted on the number line

To model 1/5 + (-4/5) on a number line, we can start by placing a point at 0 on the number line. Then we can represent 1/5 by placing another point to the right of 0, one-fifth of the distance from 0 to 1. We can represent -4/5 by placing a point to the left of 0, four-fifths of the distance from 0 to -1.

To add these two points together, we start at the point representing 1/5 and move four-fifths of the distance towards the left, since -4/5 is negative. The resulting point represents the sum of 1/5 and -4/5.

To solve this problem algebraically, we can add the two fractions together by finding a common denominator:

1/5 + (-4/5) = 1/5 - 4/5

= (1-4)/5

= -3/5

= -0.6

Learn more about number line here

brainly.com/question/17106183

#SPJ4

Jaxon is flying a kite, holding his hands a distance of 3.25 feet above the ground and
letting all the kite's string play out. He measures the angle of elevation from his hand
to the kite to be 24°. If the string from the kite to his hand is 105 feet long, how many
feet is the kite above the ground? Round your answer to the nearest tenth of a foot if
necessary.

Answers

Answer: 46.0 ft

Step-by-step explanation:

[tex]\text{sin} \ 24^o=\dfrac{x}{105}[/tex]

[tex]x=105 \ \text{sin 24}^o[/tex]

So, the distance above the ground is [tex]\text{105 sin} \ 24^o+3.25\thickapprox \boxed{46.0 \ \text{ft}}[/tex]

PLEASEEE HELP I NEED THIS ASAPPP

Answers

Answer:

perimeter=42

Area=90

Step-by-step explanation:

find the length of DEFG:

perimeter=a=b+a+b

28=10+b+10+b

28=20+2b

8=2b

4=b

finding the scale factor:

15/10=3/2

to find the perimeter of WXYZ:

the missing side:

=4x3/2

=6

perimeter=a+b+a+b

perimeter=15+6+15+6

perimeter=42

Area of WXYZ:

Area=bh

Area=15x6

Area=90

(-6, -2) (-2, 0) what is solution to system of equations?

Answers

Note that the solution of the system of equations will be (-6, -2). (Option A)

What is a system of equation?


A system of equations is a collection of two or more equations with a shared set of unknown variables. The goal is to find the values of the variables that satisfy all the equations simultaneously.

Note that System of equations is represented by two straight lines on a graph.

And solution of the system of equations is the point of intersection of these lines, because that is the point where the values from both functions satisfy all the equations simultaneously.

From the graph attached, two straight lines represent the system of equations.

And the point of intersection of these lines is the solution.

Therefore, solution of the system of equations will be (-6, -2). (Option A)

Learn more about System of Equations on:

https://brainly.com/question/9351049

#SPJ1

Full Question:

Although part of your question is missing, you might be referring to this full question:

What is the solution to the system of equations?
A (-6,2-)

B (-2, 6)

C (6,2)

D (-2,-6)​? See attached image.

A bag of oranges weighs 1.5 kg to the nearest 100 g. Complete the error interval, where x is the weight of the oranges.​

Answers

1.45kg ≤ x < 1.55kg
Hope this helps.

Blaine works for a battery manufacturing company. he wants to develop a method to test the batteries made each day to determine if they work. which method would provide the most valid results?

Answers

Random sampling 50 batteries throughout the day and testing to see if they would provide most valid results.

A sample is described as a smaller, more manageable representation of a larger group. A smaller population with characteristics of a larger one. A sample is used in statistical analysis when the population size is too large to include all participants or observations in the test.

The sample may be biased if it is taken from the first 25 batteries made each day or the last 25 batteries made each day.

The first and last battery made each day could not be an accurate representation of the total quality of the batteries produced that day if the battery manufacturing process is not consistent thought the day.

Although the battery quality can change during the day, sampling the first 50 batteries produced each day may also add bias into the sample.

The ideal course of action is to randomly select 50 batteries throughout the day, since this helps to ensure that the sample is reflective of the general calibre of batteries manufactured that day.

A more accurate image of the quality of the batteries manufactured can be obtained using this method, which is also more likely to capture any variations in battery quality over the day.

To know more about Random sampling go through:-

https://brainly.com/question/13219833

#SPJ4

Using the inverse transform method for discrete distribution, define a process for generating random variates from a binomial distribution with N 6 and p 0.4. (sixth decimal place.)

Answers

The another random variate from a binomial distribution with n=6 and p=0.4 has been generated as X=3.

The process of generating random variates from a binomial distribution using the inverse transform method for discrete distribution is done as follows:Step 1: Determine the probability mass function (pmf) of the binomial distribution with parameters n and p. For example, for a binomial distribution with n = 6 and p = 0.4, the pmf is given by:P(X=k) = (6 choose k)(0.4)^k(1-0.4)^(6-k), where k=0,1,2,3,4,5,6Step 2: Calculate the cumulative distribution function (CDF) of the binomial distribution by summing the pmf up to each value of k. The CDF is given by:F(k) = P(X ≤ k) = ΣP(X=i) for i=0 to k, where k=0,1,2,3,4,5,6Step 3: Generate a uniform random variate, U, between 0 and 1. For example, U=0.23456.Step 4: Find the smallest value of k such that F(k) ≥ U. This value of k is the random variate X. For example, if U=0.23456, then F(0)=0.10737, F(1)=0.38223, and F(2)=0.74304. Since F(1) is the smallest value of F(k) that is greater than or equal to U, X=1. Therefore, a random variate from a binomial distribution with n=6 and p=0.4 has been generated as X=1.To find another random variate from the same distribution, repeat steps 3 and 4. For example, U=0.987654, F(0)=0.10737, F(1)=0.38223, F(2)=0.74304, F(3)=0.91892, F(4)=0.98544, and F(5)=0.99856. Since F(3) is the smallest value of F(k) that is greater than or equal to U, X=3. Therefore, another random variate from a binomial distribution with n=6 and p=0.4 has been generated as X=3.

Learn more about Random

brainly.com/question/25476772

#SPJ11

Move the manilla point as close to the circle as possible so that the blue arc almost disappears keep the manilla point on the circle​

What previously learned theorem do these transformations reveal

Answers

A theorem which these transformations reveal include the following: theorem of intersecting secants.

What is the theorem of intersecting secants?

In Mathematics and Geometry, the theorem of intersecting secants states that when two lines intersect outside a given circle, the measure of the angle formed by these intersecting lines is equal to one-half (½) of the difference of the two (2) arcs it intercepts.

By applying the theorem of intersecting secants, the value of any of the angle subtended by the intersecting lines can be calculated by using the following mathematical equation:

m∠a = One-half(y – x).

m∠a = ½(y – x).

Where:

x and y represent the angles formed by the intersecting lines.

Therefore, the theorem of intersecting secants describe these set of transformations.

Read more on intersecting secants here: https://brainly.com/question/31049910

#SPJ1

question at a local ice-cream store, 210 people were surveyed on whether they preferred eating ice cream from a cone or a cup. of the 210 people surveyed, 70 were adults and 140 were children. of the responses, 150 indicated the cone as the preferred method of eating ice cream. for those surveyed, there was no association between age and preferred method of eating ice cream. which of the following tables shows the distribution of responses? responses

Answers

Based on the information provided, the table that correctly displays the data is table III.

What conditions should be met for the table to correctly display the data?To begin, the table should show the total of people surveyed was 210 people.In the table, the total of adults should be 70 and the total of children should be 140.The table should show the results between those who preferred cones vs cups are distributed between adults and children rather than cup or cone being significantly preferred by a group.

Note: The question is incomplete; below I attach the missing information:

Learn more about tables in https://brainly.com/question/10670417

#SPJ1

the diameters of ball bearings are distributed normally. the mean diameter is 58 millimeters and the standard deviation is 6 millimeters. find the probability that the diameter of a selected bearing is greater than 52 millimeters. round your answer to four decimal places.

Answers

The probability that the diameter of a selected bearing is greater than 52 millimeters is 0.8413. This can be calculated using the formula for probability of a normal distribution:

P(x > 52) = 1 - P(x ≤ 52)

P(x ≤ 52) = (52 - 58) / 6 = -1

P(x > 52) = 1 - P(-1) = 1 - 0.1587 = 0.8413.

Therefore, the probability that the diameter of a selected bearing is greater than 52 millimeters is 0.8413.

The diameters of ball bearings are distributed normally. the mean diameter is 58 millimeters and the standard deviation is 6 millimeters : https://brainly.com/question/31110857

#SPJ11

4. Assume that 4 years from now you will need $1,000. Your bank compounds interest
at an 8% annual rate. A. How much must you deposit 1 year from now to have a balance of $1,000 at
Year 4?
b. If you want to make equal payments at the end of Years 1 through 4 to
accumulate the $1,000, how large must each of the 4 payments be?
c. If your father were to offer either to make the payments calculated in part b
($221. 92) or to give you a lump sum of $750 one year from now, which would
you choose?
d. If you will have only $750 at the end of Year 1, what interest rate, compounded
annually, would you have to earn to have the necessary $1,000 at Year 4?
e. Suppose you can deposit only $186. 29 each at the end of Years 1 through 4, but
you still need $1,000 at the end of Year 4. What interest rate, with annual

Answers

A. deposit $735.03 one year from now. B. $210.48 for each of the four years. C. choose the lump sum of $750 from your father. D. Interest rate of 7.4% compounded annually. E. Interest rate of 12%.

a. To have a balance of $1,000 four years from now, you need to calculate the present value of this amount. Using the compound interest formula, we can calculate that you need to deposit $735.03 one year from now to have a balance of $1,000 four years from now.

b. If you want to make equal payments at Years 1 through 4 to accumulate the $1,000, you can use the annuity formula to calculate the required payments. The formula gives us a payment of $210.48 for each of the four years.

c. If your father were to offer either to make the payments calculated in part b or to give you a lump sum of $750 one year from now, you can compare the present value of both options. The present value of the four payments of $210.48 each, using an 8% annual interest rate, is $682.96. Therefore, you would choose the lump sum of $750 from your father.

d. If you have only $750 one year from now, you can use the compound interest formula to calculate the interest rate you need to earn to have $1,000 four years from now. Using the formula, we get an interest rate of 7.4% compounded annually.

e. If you can only deposit $186.29 each at Years 1 through 4, you can use the compound interest formula to calculate the interest rate you need to earn to have $1,000 four years from now. Using the formula, we get an interest rate of 12% compounded annually.

Learn more about compound interest here: brainly.com/question/14295570

#SPJ4

Complete question:

Assume that 4 years from now you will need $1,000. Your bank compounds interest at an 8% annual rate.

a. How much must you deposit 1 year from now to have a balance of $1,000 4 years from now?

b. If you want to make equal payments at Years 1 through 4 to accumulate the $1,000, how much each of the 4 payments be?

c. If your father were to offer either to make the payments calculated in part b or to give you a lump sum of $750 1 year from now, which would you choose?

d. If you have only $750 1 year from now, what interest rate, compounded annually, would you have to earn to have the necessary $1,000 4 years from now? 7.4%

e. Suppose you can deposit only $186.29 each at Years 1 through 4, but you still need $1,000 at Year 4. What interest rate, with annual compounding, must you seek out to achieve your goal?

Roll a fair die three times. What is the probability that it is 1 or 2 on the first roll, 3 or 4 on the second roll, or 5 or 6 on the third roll?

Answers

The probability that it is 1 or 2 on the first roll, 3 or 4 on the second roll, or 5 or 6 on the third roll when a fair die is rolled three times is 1/4.

Probability is the study of random events. It is a measure of the likelihood of an event occurring. Probability can be expressed as a decimal, a fraction, or a percentage. There are two types of probability - empirical probability and theoretical probability.

Empirical probability is calculated by conducting experiments or collecting data. It is calculated using the following formula:

Empirical probability = Number of favourable outcomes/Total number of outcomes

Theoretical probability is calculated using probability formulas. It is calculated using the following formula:

Theoretical probability = Number of favourable outcomes/Total number of possible outcomes

In the given problem, a fair die is rolled three times. We need to find the probability that it is 1 or 2 on the first roll, 3 or 4 on the second roll, or 5 or 6 on the third roll. There are 2 favourable outcomes for the first roll, 2 favourable outcomes for the second roll, and 2 favourable outcomes for the third roll.

Total number of outcomes = 6×6×6 = 216

Number of favourable outcomes = 2×2×2 = 8

Probability = Number of favourable outcomes/Total number of outcomes

Probability = 8/216

Probability = 1/27

Therefore, the probability that it is 1 or 2 on the first roll, 3 or 4 on the second roll, or 5 or 6 on the third roll when a fair die is rolled three times is 1/4.

You can learn more about probability at: brainly.com/question/11234923

#SPJ11

At a party to celebrate a successful school play, the drama club bought 999 large pizzas. Each pizza had sss slices. All together, there were 727272 slices of pizza for the club to share.
Write an equation to describe this situation.
How many slices does each pizza have?

Answers

Answer:

Step-by-step explanation:

Let's use the variable "s" to represent the number of slices in each pizza.

The drama club bought 999 large pizzas, and each pizza had s slices. So, the total number of pizza slices is given by the product of 999 and s, which can be written as:

999s = 727272

To solve for s, we can divide both sides of the equation by 999:

s = 727272/999

Using a calculator, we can simplify this fraction to:

s ≈ 728.73

Therefore, each pizza has approximately 728 slices. Note that this is an unusual number of slices for a pizza, so it's possible that the problem is not intended to be taken literally, but rather as a math puzzle or word problem.

Answer:

Each pizza has 729 slices.

Step-by-step explanation:

j by 9 and then multiply the quotient by 2

Answers

The sequence of the questions is [tex]J/9[/tex] when you multiply [tex]J[/tex] by [tex]9[/tex] and the quotient by [tex]2[/tex].

What in arithmetic is a quotient?

In this example, the number being divided (15) is known as the result, and the number being divided by (3 in this instance) is known as the divisor. The quotient is the outcome of the division.

Is quotient the same as answer?

The result of dividing two integers is known as the quotient. Six divided into two results in the number three. Latin's "how many times" is the meaning of the word "quotient." It makes perfect sense: by dividing two numbers, you can determine "how many time" the two digits enters the first.

[tex]2 * (J/9)[/tex]

This means you first divide [tex]J[/tex] by [tex]9[/tex] to get the quotient, and then multiply the quotient by [tex]2[/tex]. So the order of operations is:

[tex]J/9[/tex]

Multiply the quotient by [tex]2[/tex]

For example, if [tex]J = 45[/tex], then:

[tex]J/9 = 45/9 = 5[/tex]

[tex]2 * (J/9) = 2 * 5 = 10[/tex]

So the result of multiplying [tex]J[/tex] by [tex]9[/tex] and then multiplying the quotient by [tex]2[/tex], when [tex]J = 45[/tex], is [tex]10[/tex].

To know more about quotient visit:

https://brainly.com/question/16675988

#SPJ1

Frannie has $120. She spends 30% of the money she has on a ticket to the theater. How much did Frannie pay for the theater ticket?

Answers

Answer:

36

Step-by-step explanation:

PxW=p

30x120=p

3600=p

x100%

$36=p

(overpriced ticket of you ask me lol)

the nutty professor sells cashews for $7.70 per pound and brazil nuts for $4.80 per pound. how much of each type should be used to make a 27 pound mixture that sells for $6.41 per pound?

Answers

The amount that each type would be 11.87 lbs of cashews and 15.13 lbs of brazil nuts

1. First, find the total cost of 27 lbs of the mixture: 27 lbs x $6.41/lb = $171.07.
2. Next, find the cost of cashews and brazil nuts in the mixture. Cashews cost $7.70/lb and brazil nuts cost $4.80/lb.
3. Subtract the cost of the brazil nuts from the total cost of the mixture: $171.07 - (27 lbs x $4.80/lb) = $105.27.
4. Divide the cost of the cashews ($105.27) by the cost of one pound of cashews ($7.70): $105.27/$7.70 = 13.66 lbs.
5. Subtract the number of pounds of cashews (13.66) from the total pounds of the mixture (27) to find the number of pounds of brazil nuts: 27 - 13.66 = 15.13 lbs.
6. Therefore, the mixture should contain 11.87 lbs of cashews and 15.13 lbs of brazil nuts.

See more about total cost at: https://brainly.com/question/5168855

#SPJ11

compute the zeros of the polynomial 4x2 - 4x - 8

Answers

Answer:

(2, 0) and (-1, 0)

Step-by-step explanation:

[tex]4x^2 - 4x - 8 = 0 \text{ // Divide by 4} \\x^2 - x - 2 = 0\\\\x_{1, 2} = \frac{-(-1) \pm \sqrt{(-1)^2 - 4 \times 1 \times (-2)}}{2\times1}\\\\x_{1, 2} = \frac{1 \pm \sqrt{1 + 8}}2\\\\x_{1, 2} = \frac{1 \pm \sqrt9}2\\\\x_{1, 2} = \frac{1 \pm 3}2\\\\x_1 = \frac{1 + 3}2 = \frac42 = 2\\\\x_2 = \frac{1 - 3}2 = \frac{-2}2 = -1[/tex]

Therefore, the zeroes are (2, 0) and (-1, 0).

how to calculate the product of two random variable that follows normal distribution with mean 0 and variance 1

Answers

To calculate the product of two random variables that follows the normal distribution with mean 0 and variance 1 by using the covariance formula

Cov(X, Y) = E[XY] - E[X]E[Y] = E[XY] - 0 = E[XY]

Given that two random variables follow a normal distribution with mean 0 and variance 1.

Let X and Y be two independent normal random variables such that X ~ N(0,1) and Y ~ N(0,1)

Now, The expected value of the product of two random variables is given by;

E[XY] = E[X]E[Y] + Cov(X,Y)

Where E[X] and E[Y] are the means of the two random variables X and Y respectively.

Cov(X, Y) is the covariance between the two random variables, which can be calculated using the formula;

Cov(X,Y) = E[XY] - E[X]E[Y]

Now, E[X] = E[Y] = 0 as both have a mean of 0.

Cov(X, Y) = E[XY] - E[X]E[Y]

⇒ E[XY] = the expected value of the product of X and Y.

As X and Y are independent, their covariance will be zero, which implies;

Cov(X, Y) = E[XY] - E[X]E[Y] = E[XY] - 0 = E[XY]

Thus, we can calculate the product of two random variables that follow a normal distribution with mean 0 and variance 1 using the above formula for covariance.

To know more about the "covariance": https://brainly.com/question/21287720

#SPJ11

What is result of following operation(4623. 56)10+ (110011. 11)2whare (110011. 11(2 mean that 110011. 11as a number express in base 2​

Answers

The given numbers are in decimal and binary system and the final result of the given operation is [tex](4675.31)_{10}[/tex].

A binary integer (base-2) is converted to an equivalent decimal number using the binary to decimal conversion formula. (base-10). In mathematics, integers are expressed using a number system. It is a method to display numerical data. The four various numeral systems are as follows:

System of Binary Numbers (Base-2)
system of octal numbers (Base-8)
System of Decimal Numbers (Base-10)
System of Hexadecimal Numbers (Base-16).
We are the two numbers:-

[tex](4623.56)_{10} , (110011.11)_{2}[/tex]

these are in decimal and binary system respectively.

now, we will express them in same system ( here we choose decimal system).

[tex](110011.11)_{2} = (2^{5} + 2^{4} + 0 + 0 + 2^{1} + 2^{0} + 2^{-1} + 2^{-2} )_{10} \\= (2^{5}+2^{4}+0*2^{4}+0*2^{3}+2^{1}+2^{0}+2^{-1}+2^{-2})_{10} \\= (32+16+2+1+0.5+0.25)_{10} \\= (51.75)_{10}[/tex]

Now, addition is done below:-

4623.56+51.75= 4675.31.

hence, the final result of the given operation= [tex](4675.31)_{10}[/tex]

To know more about binary system go through:-

https://brainly.com/question/16612919

#SPJ4

I think it’s to do with Pythagoras theorm

Answers

Split the triangle in half.

8.6cm is the hypotenuse and 8.6/2 is the length of base.

The Pythagorean theorem: a^2+b^2=c^2
So that means 8.6^2-4.3^2=height

= 73.96 - 18.49 = 55.47

So the square root of 55.47 is the height of the triangle.

The area of a triangle is h * b/2

So the base is 8.6 and the height is the square root of 55.47. 8.6/2=4.3

So 4.3 times the square root of 55.47 is the area.

Mrs Smith walks a half a mile a day after work. She works five days a week. How many yards will she have walked for the week by Friday morning?

Answers

The distance Mrs. Smith covers is 3520 yards during the duration of the week by Friday morning.

One week has seven days in total.

Mrs. Smith walks half a mile each day after work, she walks a total of

0.5 miles/ day × 7 days/ week = 3.5 miles/ week

Now, if we calculate the distance on Friday morning, she must have walked four times till Friday morning since she has to walk after her work.

Therefore,

0.5 miles/ day × 4 days = 2 miles

To convert miles to yards, we can use the fact that there are 1760 yards in one mile:

2 miles/week × 1760 yards/mile = 3520 yards/week

Therefore, by Friday morning, Mrs. Smith will have walked 3520 yards.

To learn more about the distance at

https://brainly.com/question/26264531

#SPJ4

Write each equation in slope-intercept form. Identify the slope and y-intercept.

x - 3y = 12

*Work must be shown.*​

Answers

Answer:

slope is 1/3

y-intercept is -4

Step-by-step explanation:

x - 3y = 12

3y = x - 12

y = 1/3x - 4

according to y = mx + b, m is slope and b is y-intercept

slope is 1/3

y-intercept is -4

two people standing at different locations are looking at a tall building. person a angle of elevation to the building is 35 degrees. person b angle of elevation is 77 degrees. the building is 8 miles away from person b. how far away is person a from the building?

Answers

Therefore, Person A is approximately 95.17 miles away from the building.

To find out how far person A is from the building, we'll need to use trigonometry. The diagram below shows the situation.

Given that Person A's angle of elevation to the building is 35 degrees, we'll let angle BAC be 35 degrees.

Similarly, since Person B's angle of elevation is 77 degrees, we'll let angle ABC be 77 degrees. We'll also let AB be x, the distance from Person A to the building, and BC be 8 miles, the distance from Person B to the building.

First, we'll use the tangent function to find the height of the building. In triangle ABC, tan(77) = height/8. Solving for the height, we get:

height = 8tan(77) ≈ 61.23 miles.

Next, we'll use the tangent function again to find x. In triangle ABC, tan(35) = height/x + 8. Solving for x, we get:

x = (height)/(tan(35)) - 8
≈ 103.17 miles - 8
≈ 95.17 miles.
for such more questions on  elevation

https://brainly.com/question/25748640

#SPJ11

A shipping company is packing a box with one cubic centimeters blocks. The box is 14 centimeters long. 12 centimeters wide and 16 centimeters high. How many one cubic centimeter blocks will fill the box ?

Answers

The shipping company will need 2,688 one cubic centimeter blocks to fill the box.

The volume of the box can be calculated as:

Volume = length x width x height

Volume = 14 cm x 12 cm x 16 cm

Volume = 2,688 cubic cm

Since each block is also one cubic cm in volume, we can simply divide the total volume of the box by the volume of one block to find the number of blocks needed to fill the box:

Number of blocks = Volume of box / Volume of one block

Number of blocks = 2,688 cubic cm / 1 cubic cm

Number of blocks = 2,688

Learn more about volume here

brainly.com/question/23952628

#SPJ4

HELP PLEASE … Assuming the input of energy continues for another 2.5 seconds, where will the particle be?

A) cannot be determined
B) positive maximum
C) negative maximum
D) equilibrium

Answers

Answer:

To determine the position of a particle given the input of energy, we need to know the type of energy input and the initial position and velocity of the particle. Without this information, we cannot determine the position of the particle after 2.5 seconds.

Therefore, the answer is A) cannot be determined.

Step-by-step explanation:

ABOVE

Caris has a carton of 12 eggs, two of which have brown shells while the rest have white shells. Caris randomly chooses a brown egg from the carton. Which of the following statements is true? If she rejects this egg, returns it to the carton, and randomly picks again, these will be dependent events. If she uses this egg in a recipe and picks another one from the carton, these will be dependent events. Whether or not these are dependent or independent events depends on what color egg Caris chooses next. If she uses this egg in a recipe and picks another one from the carton, these will be independent events.

Answers

Answer:

Step-by-step explanation:

i think you have to times it

in august 2012, tropical storm isaac formed in the caribbean and was headed for the gulf of mexico. there was an initial probability of .69 that isaac would become a hurricane by the time it reached the gulf of mexico (national hurricane center website, august 21, 2012). a. what was the probability that isaac would not become a hurricane but remain a tropical storm when it reached the gulf of mexico (to 2 decimals)? b. two days later, the national hurricane center projected the path of isaac would pass directly over cuba before reaching the gulf of mexico. hurricanes that reach the gulf of mexico have a .08 probability of having passed over cuba. tropical storms that reach the gulf of mexico have a .20 probability of having passed over cuba. how did passing over cuba alter the probability that isaac would become a hurricane by the time it reached the gulf of mexico? use the above probabilities to answer this question. p(c|h) (to 2 decimals) p(c|t) (to 2 decimals) p(h|c) (to 4 decimals) c. what happens to the probability of becoming a hurricane when a tropical storm passes over a landmass such as cuba? select (to 2 decimals) to (to 4 decimals).\

Answers

By using  Bayes' theorem in probability.

a) The probability that isaac would not become a hurricane but remain a tropical storm when it reached the gulf of mexico is 0.31.

b) P(H|c) = 0.4493

c) It can be observed that the likelihood of a tropical storm developing into a hurricane decreases slightly when it passes over Cuba. This can be inferred from the fact that the conditional probability of a hurricane forming given that the storm has passed over Cuba (P(H|c) = 0.4493) is lower than the initial probability of 0.69.

a) The probability that Isaac would not become a hurricane but remain a tropical storm when it reached the Gulf of Mexico is:

P(not hurricane) = 1 - P(hurricane) = 1 - 0.69 = 0.31

So the probability is 0.31 (to 2 decimals).

b) We need to use Bayes' theorem to calculate the probabilities:

P(c|H) = P(H|c) * P(c) / P(H)

P(c|T) = P(T|c) * P(c) / P(T)

where c denotes passing over Cuba, H denotes becoming a hurricane, and T denotes remaining a tropical storm.

From the problem, we have:

P(H) = 0.69

P(T) = 1 - P(H) = 0.31

P(c|H) = 0.08

P(c|T) = 0.20

To calculate P(c), we need to use the law of total probability:

P(c) = P(c|H) * P(H) + P(c|T) * P(T)

= 0.08 * 0.69 + 0.20 * 0.31

= 0.1228

Now we can calculate P(c|H) and P(c|T):

P(c|H) = 0.08 * 0.69 / 0.1228

= 0.4493 (to 2 decimals)

P(c|T) = 0.20 * 0.31 / 0.1228

= 0.5065 (to 2 decimals)

To calculate P(H|c), we use Bayes' theorem again:

P(H|c) = P(c|H) * P(H) / P(c)

= 0.08 * 0.69 / 0.1228

= 0.4493 (to 4 decimals)

c) Passing over a landmass such as Cuba can alter the probability of becoming a hurricane because it can either enhance or weaken the storm. In this case, we can see that the probability of becoming a hurricane is actually slightly lower when a tropical storm passes over Cuba, as P(H|c) = 0.4493 is lower than the initial probability of 0.69. However, it is important to note that this is just one example and the effect of passing over a landmass can vary depending on many factors.

To know about "Bayes' theorem": https://brainly.com/question/14989160

#SPJ11

Keilantra and Samantha work at a dry cleaners ironing shirts. Keilantra can iron 30 shirts per hour, and Samantha can iron 15 shirts per hour. Keilantra and Samantha worked a combined 11 hours and ironed 240 shirts. Graphically solve a system of equations in order to determine the number of hours Keilantra worked, x, and the number hours Samantha worked, y.

Answers

Answer: Kelinatra (x) worked 5 hours and Samantha (y) worked 6 hours

Step-by-step explanation:

     We will use the variables x and y the question provides. We know the time worked by each person added together will equal the combined total time. We can write an equation to show this using addition.

        x + y = 11 hours

     Next, we know that Keilantra ironed 30 per hour, Samantha ironed 15 per hour and that they ironed 240 shirts. We can write another equation to represent this using addition and multiplication.

     30x + 15y = 240

     Next, we will graph these two equations. See attached. The solution is the point of intersection written as (x, y). This is (5, 6) meaning that Kelinatra (x) worked 5 hours and Samantha (y) worked 6 hours.

Triangle lmn will be dilated with respect to the origin by a scale factor of 1/2

what are the new coordinates of L’M’N’

Answers

The triangle LMN, with vertices L(6, −8), M(4, −4), and N(−12, 2), dilated with respect to the origin by a scale factor of 1/2, results in triangle L'M'N', with vertices L'(3, -4), M'(2, -2), and N'(-6, 1)

To dilate a triangle with respect to the origin, we need to multiply the coordinates of each vertex by the scale factor. In this case, the scale factor is 1/2, so we multiply each coordinate by 1/2.

The coordinates of L' are obtained by multiplying the coordinates of L by 1/2:

L'((1/2)6, (1/2)(-8)) = (3, -4)

The coordinates of M' are obtained by multiplying the coordinates of M by 1/2:

M'((1/2)4, (1/2)(-4)) = (2, -2)

The coordinates of N' are obtained by multiplying the coordinates of N by scale factor 1/2:

N'((1/2)×(-12), (1/2)×2) = (-6, 1)

Learn more about dilation here

brainly.com/question/26155897

#SPJ4

The given question is incomplete, the complete question is:

Triangle LMN with vertices L(6, −8), M(4, −4), and N(−12, 2) is dilated with respect to origin by a scale factor of 2 to obtain triangle L′M′N′. What are the new coordinates of  L′M′N′ ?

Other Questions
Two different types of staff work at a halfway house. One group is primarily involved in activities that are custody oriented, whereas the case managers and counselors take care of treatment and rehabilitation. T/F ASTRONOMY!!During her presentation on exoplanets, Johana explains to the class that while Proxima-b and TRAPPIST-1e maypotentially be able to support life, with each of these planets, one side of the planet always faces the sun, making thatside perpetually hot and the dark side eternally cool. What term does Michelle use to describe this?extinguishabletidally unlockedtidally lockedbipolar many tests to distinguish aldehydes and ketones involve the addition of an oxidant. only choose... can be easily oxidized because there is choose... next to the carbonyl and oxidation does not require choose... (a) A roller-coaster car has a total mass (including passengers) of 505 kg. Sitting in the car is a passenger with a mass of 52.0 kg. The car reaches the lowest point of a circular arc in the track, point A in the figure below, moving at a speed of 14.0 m/s. The radius of the arc is r, = 24.0 m. What is the magnitude (in N) and direction of the force that the seat exerts on the passenger at point A? magnitude direction Select-- v (b) What If? If the car has the same speed at point A as in part (a), what would the radius (in m) of the track have to be for the force of the seat on the passenger at this point to be three times the passenger's weight? 17- 20: In an August 2012 Gallup survey of 1,012 randomly selected U.S. adults (age 18 and over), 53% said that they were dissatisfied with the quality of education students receive in kindergarten through grade 12. They also report that the "margin of sampling error is plus or minus 4%."17. What is the population of interest?18. What is the sample being used?19. What is the population parameter of interest, and what is the correct notation for this parameter? What is the relevant statistic?20. Find an interval estimate for the parameter of interest. Interpret it in terms of dissatisfaction in the quality of education students receive. Use two decimal places in your answer. Using the inverse transform method for discrete distribution, define a process for generating random variates from a binomial distribution with N 6 and p 0.4. (sixth decimal place.) Determine the slope from the table given below. Mrs Smith walks a half a mile a day after work. She works five days a week. How many yards will she have walked for the week by Friday morning? changes in agriculture may have contributed to global changes in temperature. 4) The Dallas, Phoenix, and Tulsa reported income of $140,000 from their partnership for the year ended December 31, 2019. Profits and losses are to be distributed as follows:Dallas Phoenix Tulsa Salaries $35,000 $25,000 $20,000 Bonus 15% -- -- Profit and Loss sharing 60% 30% 10%How should partnership net income for 2019 be allocated to Dallas, Phoenix, and Tulsa?Dallas, Phoenix, Tulsa A) $79,000 $36,700 $24,300 B) $79,400 $36,700 $23,900 C) $55,200 $57,000 $27,800 D) $45,200 $40,000 $44,800 suppose you get a splinter in your epidermis which kind of epidermal cell would trigger an immune reaction g In which of the following groups of trade blocs does most of today's world trade take place?A) Middle East, China, and IndiaB) North America, Africa, and CanadaC) Western Europe, Asia, and the AmericasD) Saudi Arabia, Western Europe, and the Gulf what type of hearing aid fits completely inside the external auditory canal? (01.05 MC)Danielle is writing a thesis statement about the impact of indirect characterization on the development of the plot. Read her thesis statement and answer the question that follows.In "The Ascent" by Ron Rash, there are many things that the author does with the people in the story to make the plot go.What should Danielle most focus on when revising her thesis statement for formal language? Which of the following quotations best represents the thesis statement of the passage?"But reflecting persons perceived that when society is itself the tyrantsociety collectively over the separate individuals who compose itits means of tyrannising are not restricted to the acts which it may do by the hands of its political functionaries." (paragraph 1, sentence 2)"Protection, therefore, against the tyranny of the magistrate is not enough; there needs protection also against the tyranny of the prevailing opinion and feeling . .." (paragraph 1, sentence 4)"But though this proposition is not likely to be contested in general terms, the practical question, where to place the limithow to make the fitting adjustment between individual independence and social controlis a subject on which nearly everything remains to be done." (paragraph 2, sentence 1)"All that makes existence valuable to anyone depends on the enforcement of restraints upon the actions of other people." (paragraph 2, sentence 2)"No two ages, and scarcely any two countries, have decided it alike; and the decision of one age or country is a wonder to another." (paragraph 2, sentence 5) Pawprints Paint recently went public in a best efforts offering. The company offered 85,000 shares of stock for sale at an offer price of $41 per share. The administrative costs associated with the offering were $325,000 and the underwriter's spread was 8 percent. After completing their sales efforts. The underwriters determined that they sold a total of 81,200 shares. What were the net proceeds to the company?Multiple ChoiceA. $2,737,864B. $2,556,200C. $3,062,864D. $2. 412,864E. $2,881,200 Your local barista recommends a new Thai restaurant that just opened down the street. This exchange demonstrates the usefulness of __________ in relationships. facework empathy weak ties relational listening Complete the synthesis by determining the set of reactions and the synthetic intermediate needed to convert the given alkyl halide to the primary amine. Drag the appropriate labels to their respective targets Hints NH HNNH2 1) HCrO 2) Hyo H2. Raney Ni H,NOH NaN3 excess NH NT trace acid DMF Br NH2 the nutty professor sells cashews for $7.70 per pound and brazil nuts for $4.80 per pound. how much of each type should be used to make a 27 pound mixture that sells for $6.41 per pound? Flextime is a system that allows employees some freedom to choose which hours to work as long as they:- work schedule that gives employees some freedom- work the required number of hours- complete their assigned tasks